PT 57 Game 4 #21 Forum

Prepare for the LSAT or discuss it with others in this forum.
Post Reply
KylieMorrison

New
Posts: 29
Joined: Sat Sep 11, 2010 2:22 pm

PT 57 Game 4 #21

Post by KylieMorrison » Thu Sep 30, 2010 8:42 pm

I am confused about what exactly this question is asking. If exactly 2 grants are awarded in just one of the 4 quarters then which one could be true? Is it wanting T and Y to be together in 1 quarter or T and Y to be the only ones in a quarter?

Could you not do:
T M Y M
W W
1 2 3 4

That would mean T and Y are awarded in 1 quarter each and that makes C possible.
If someone could clear this up I would appreaciate it. Thanks.

User avatar
Cromartie

Bronze
Posts: 200
Joined: Wed Aug 18, 2010 12:27 pm

Re: PT 57 Game 4 #21

Post by Cromartie » Thu Sep 30, 2010 9:05 pm

Look at your example. 2 grants are awarded in 2 quarters (Q1 and Q2), which violates the requirement. You have to come up with a 2-1-1-1 (in no particular order) distribution that complies with all the other rules.

KylieMorrison

New
Posts: 29
Joined: Sat Sep 11, 2010 2:22 pm

Re: PT 57 Game 4 #21

Post by KylieMorrison » Thu Sep 30, 2010 9:40 pm

Hmmm I guess I'm still confused. Even if 2 grants are awarded in 2 quarters there are still 2 grants awarded in 1 quarter. I just think I'm not understand what the question is asking. When it is saying 2 grants awarded in just one quarter is it saying that only 1 quarter can have 2 grants or at least 2 of the grants can only be in one quarter?

User avatar
Cromartie

Bronze
Posts: 200
Joined: Wed Aug 18, 2010 12:27 pm

Re: PT 57 Game 4 #21

Post by Cromartie » Thu Sep 30, 2010 10:00 pm

KylieMorrison wrote:Hmmm I guess I'm still confused. Even if 2 grants are awarded in 2 quarters there are still 2 grants awarded in 1 quarter. I just think I'm not understand what the question is asking. When it is saying 2 grants awarded in just one quarter is it saying that only 1 quarter can have 2 grants or at least 2 of the grants can only be in one quarter?
Only 1 of the 4 quarters has exactly 2 grants. None of the other quarters can have exactly 2 grants. Hence, the other quarters must have either 1 or more than 2 (i.e., 3 or 4) grants. Having 3 or 4 grants in another quarter will lead to a violation of the general condition that there are at most 6 grants awarded in the year (2+3+1+1 = 7). Hence, all the other 3 quarters must have only 1 grant each, which leads to the 2-1-1-1 distribution.

Hope that makes sense.

KylieMorrison

New
Posts: 29
Joined: Sat Sep 11, 2010 2:22 pm

Re: PT 57 Game 4 #21

Post by KylieMorrison » Fri Oct 01, 2010 9:46 am

Yeah I understand now. I was making it way more difficult than it really was. Thank you!

Want to continue reading?

Register now to search topics and post comments!

Absolutely FREE!


Post Reply

Return to “LSAT Prep and Discussion Forum”